There are four white socks and six black socks in a drawer what is the probability of selecting a white sock not replacing it and then selecting another white sock

Answers

Answer 1

Answer:

Step-by-step explanation:

Add all the socks in the drawer to get a denominator

12

blue socks can be picked out of the

36

total socks, so

12

36

Then you take away one sock from the top and bottom, so you now have

11

35

Multiply the two numerators (

12

and

11

), then multiply the denominators (

36

and

35

), then divide the numerator by the denominator,

132

1260

, to get your answer.

Answer 2

Answer:

6/10

Step-by-step explanation:

1st step: 4+6=10


Related Questions

If you vertically compress the square root parent function by a factor of 1/3, what is the equation of the new function?

Answers

9514 1404 393

Answer:

  y = (1/3)√x

Step-by-step explanation:

Vertical scaling of a function is accomplished by multiplying the function by the scale factor. If you want to scale the square root function by a factor of 1/3, then the scaled function is ...

  y = (1/3)√x

The diagram shows a right-angled triangle.
xo
26 cm
17 cm
Find the size of angle x.
Give your answer correct to 1 decimal place.

Answers

Answer:

Diagram? I don't see a diagram.

Where is the diagram?

Step-by-step explanation:

Let V be the set of all 3x3 matrices with Real number entries, with the usual definitions of scalar multiplication and vector addition. Consider whether V is a vector space over C. Mark all true statements (there may be more than one).

a. The scalar closure axiom is satisfied
b. The additive inverse axiom is not satisfied
c The additive inverse axiom is satisfied
d. The additive closure axiom is not satisfied
e. The scalar closure axiom is not satisfied
f. The additive closure axiom is satsified
g. V is not a vector space over C
h. V is a vector space over C
i. The zero axiom is satisfied
j. The zero axiom is not satisfied

Answers

Answer:

the Scalar Closure axiom is not satisfiedV is not a Vector Space of CThe Additive Closure axiom is satisfied.

Step-by-step explanation:

According to the Question,

Given That, Let V be the set of all 3x3 matrices with Real number entries, with the usual definitions of scalar multiplication and vector addition. Consider whether V is a vector space over C.For V is a vector space over C and V is Set of 3x3 Matrices with Real entries.

Then, For any u,w ∈ V ⇒ u+w ∈ V

And u∈V and z∈C ⇒ z u ∈ V

So, If we take any z= i ∈ C

and V be any 3x3 matrices with Real entrices.

then, z,v ∉ V  ∴z,v Has Complex entries

So, the Scalar Closure axiom is not satisfied

Hence, V is not a Vector Space of C

Any u,w ∈ W ⇒ u+w ∈ V

So, The Additive Closure axiom is satisfied.

If f(x) = x -2 and g(x) = 2x – 6, then g(4)/f(3) =​

Answers

Answer:

Step-by-step explanation:

(2×4-6)/(3-2)=2

Answer:

[tex]{ \tt{f(x) = x - 2}} \\ { \bf{f(3) = 3 - 2 = 1}} \\ \\ { \tt{g(x) = 2x - 6}} \\ { \bf{g(4) = 2(4) - 6 = 2}} \\ \\ { \boxed{ \tt{ \frac{g(4)}{f(3)} = \frac{2}{1} = 2}}}[/tex]

 Marsha has a bag that contains 4 green marbles, 8 yellow marbles , and 20 red marbles . If she chooses one marble from the bag, what is the probability that the marble is not red?

PLEASE HELP IF YOURE GOOD AT GEOMETRY!!

Answers

Answer:

C. 3/8

HOPE THIS HELPS :)

Answer:

c. 3/8

Step-by-step explanation:

first you need the denomerator by adding all marbles together which equals 32. now for the munerator you need the sum of the green and yellow marbles. this equals 12. so your fraction is 12/32. next we simplify. we can divide both numbers by 4. getting us a fraction of 3/8.

How much is 12,856 ounces in pounds ?

Answers

Answer : 803.5 pounds

There are 16 ounces in a pound, so just divide 12,856 by 16, giving you 803.5.

2 divided by 0.75 full divison work i dont just need the answer​

Answers

Answer:

0.375

Step-by-step explanation:

Check the picture below.

whenever we do division of decimals, we have to mind how many decimals are there on each amount, the dividend as well as the divisor, that way we pad with zeros the other amount accordingly whilst losing the dot, for example, to say divide 3 by 0.123, 3 has no decimals, whilst 0.123 has three decimals, so we can just divide 3000 by 0123, so dividing 3 by 0.123 is the same as dividing 3000 by 123.  Another example, if we were to divide say 23.761 by 555.89331, the dividend has 3 decimals, that means 3 zeros the other way, the divisor has 5 decimals, that means 5 zeros the other way while losing the dots, so we'd end up dividing 2376100000 by 55589331000, which we can simplify to just 2376100 by 5589331, as you can see in the picture in this case.

A credit card company uses these rules to calculate the minimum amount owed: For a bill of less than $100, the entire amount is due. For a bill of at least $100 but less than $500, the minimum due is $100. For a bill of at least $500 but less than $1,000, the minimum due is $300. For a bill of $1,000 or more, the minimum due is $500. Which graph shows the minimum amount due for a credit amount of x (given that the credit limit is $2,000).

Answers

1000 is the right answer

How many side of the triangle are congruent? Explain.

A) 0
B) 2
C)3
D) not enough information given

Answers

Answer:

Option B

Step-by-step explanation:

In the given triangle,

Two sides of the triangle measure 2 cm and one side measures 3 cm.

Therefore, in this triangle two sides measuring 2 cm are congruent.

Option B is the correct option.

Find a power series for the function, centered at c, and determine the interval of convergence. f(x) = 9 3x + 2 , c = 6

Answers

Answer:

[tex]\frac{9}{3x + 2} = 1 - \frac{1}{3}(x - \frac{7}{3}) + \frac{1}{9}(x - \frac{7}{3})^2 - \frac{1}{27}(x - \frac{7}{3})^3 ........[/tex]

The interval of convergence is:[tex](-\frac{2}{3},\frac{16}{3})[/tex]

Step-by-step explanation:

Given

[tex]f(x)= \frac{9}{3x+ 2}[/tex]

[tex]c = 6[/tex]

The geometric series centered at c is of the form:

[tex]\frac{a}{1 - (r - c)} = \sum\limits^{\infty}_{n=0}a(r - c)^n, |r - c| < 1.[/tex]

Where:

[tex]a \to[/tex] first term

[tex]r - c \to[/tex] common ratio

We have to write

[tex]f(x)= \frac{9}{3x+ 2}[/tex]

In the following form:

[tex]\frac{a}{1 - r}[/tex]

So, we have:

[tex]f(x)= \frac{9}{3x+ 2}[/tex]

Rewrite as:

[tex]f(x) = \frac{9}{3x - 18 + 18 +2}[/tex]

[tex]f(x) = \frac{9}{3x - 18 + 20}[/tex]

Factorize

[tex]f(x) = \frac{1}{\frac{1}{9}(3x + 2)}[/tex]

Open bracket

[tex]f(x) = \frac{1}{\frac{1}{3}x + \frac{2}{9}}[/tex]

Rewrite as:

[tex]f(x) = \frac{1}{1- 1 + \frac{1}{3}x + \frac{2}{9}}[/tex]

Collect like terms

[tex]f(x) = \frac{1}{1 + \frac{1}{3}x + \frac{2}{9}- 1}[/tex]

Take LCM

[tex]f(x) = \frac{1}{1 + \frac{1}{3}x + \frac{2-9}{9}}[/tex]

[tex]f(x) = \frac{1}{1 + \frac{1}{3}x - \frac{7}{9}}[/tex]

So, we have:

[tex]f(x) = \frac{1}{1 -(- \frac{1}{3}x + \frac{7}{9})}[/tex]

By comparison with: [tex]\frac{a}{1 - r}[/tex]

[tex]a = 1[/tex]

[tex]r = -\frac{1}{3}x + \frac{7}{9}[/tex]

[tex]r = -\frac{1}{3}(x - \frac{7}{3})[/tex]

At c = 6, we have:

[tex]r = -\frac{1}{3}(x - \frac{7}{3}+6-6)[/tex]

Take LCM

[tex]r = -\frac{1}{3}(x + \frac{-7+18}{3}+6-6)[/tex]

r = -\frac{1}{3}(x + \frac{11}{3}+6-6)

So, the power series becomes:

[tex]\frac{9}{3x + 2} = \sum\limits^{\infty}_{n=0}ar^n[/tex]

Substitute 1 for a

[tex]\frac{9}{3x + 2} = \sum\limits^{\infty}_{n=0}1*r^n[/tex]

[tex]\frac{9}{3x + 2} = \sum\limits^{\infty}_{n=0}r^n[/tex]

Substitute the expression for r

[tex]\frac{9}{3x + 2} = \sum\limits^{\infty}_{n=0}(-\frac{1}{3}(x - \frac{7}{3}))^n[/tex]

Expand

[tex]\frac{9}{3x + 2} = \sum\limits^{\infty}_{n=0}[(-\frac{1}{3})^n* (x - \frac{7}{3})^n][/tex]

Further expand:

[tex]\frac{9}{3x + 2} = 1 - \frac{1}{3}(x - \frac{7}{3}) + \frac{1}{9}(x - \frac{7}{3})^2 - \frac{1}{27}(x - \frac{7}{3})^3 ................[/tex]

The power series converges when:

[tex]\frac{1}{3}|x - \frac{7}{3}| < 1[/tex]

Multiply both sides by 3

[tex]|x - \frac{7}{3}| <3[/tex]

Expand the absolute inequality

[tex]-3 < x - \frac{7}{3} <3[/tex]

Solve for x

[tex]\frac{7}{3} -3 < x <3+\frac{7}{3}[/tex]

Take LCM

[tex]\frac{7-9}{3} < x <\frac{9+7}{3}[/tex]

[tex]-\frac{2}{3} < x <\frac{16}{3}[/tex]

The interval of convergence is:[tex](-\frac{2}{3},\frac{16}{3})[/tex]

Find (x,y) such that (8,2)(x,y) = (28,22)

Answers

Answer:

(x,y) = (3.5, 11)

Step-by-step explanation:

(8,2)(x,y) = (28,22)

Dot product, so:

[tex](8,2)(x,y) = (8x,2y) = (28,22)[/tex]

Then

[tex]8x = 28[/tex]

[tex]x = \frac{28}{8} = 3.5[/tex]

And

[tex]2y = 22[/tex]

[tex]y = \frac{22}{2} = 11[/tex]

So

(x,y) = (3.5, 11)

11x+7y=17
solve for y

Answers

[tex]\implies {\blue {\boxed {\boxed {\purple {\sf {\: y = \frac{17 - 11x}{7} }}}}}}[/tex]

[tex]\large\mathfrak{{\pmb{\underline{\red{Step-by-step\:explanation}}{\red{:}}}}}[/tex]

[tex]\\11x + 7y = 17[/tex]

[tex] \\➺ \: 7y = 17 - 11x[/tex]

[tex]\\➺ \: y = \frac{17 - 11x}{7} [/tex]

[tex]\bold{ \green{ \star{ \orange{Mystique35}}}}⋆[/tex]

Which rules of exponents will be used to evallate this expression? Select three options.

Answers

I need the answer choices and the expression you’re referring to

PLEASE HELP ME ASAP!​

Answers

Answer:

A

Step-by-step explanation:

A


This is the correct answer

A person walks 1/6 mile in 1/18 hour.

The person's speed is _ miles per hour.

Answers

This Is What I Got!

Hope This Helps! :)

Have A Good Day!!

And If You Can I Wouldn't Mind A Brainliest! :))

Answer:

Divide 1/6 miles to 1/12hour since u wanna find our miles per hour

So it’ll be : 1/6 / 1/12

= 1/6 x 12/1

= 2 miles

Help me with this answer I don’t it

Answers

Answer:

f(-2) = g(-2) this is the answer

A geneticist conducts an experiment with peas, one sample of offspring consisted of 450 green peas and 157 yellow peas. Based on these results, estimate the probability of getting an offspring pea that is green.

Answers

Answer: 0.738

Step-by-step explanation:

11. Mendelian Genetics. When Mendel conducted his famous genetics experiments with peas, one sample of offspring consisted of 428 green peas and 152 yellow peas. Based on those results, estimate the probability of getting an offspring pea that is green. Is the result reasonably close to the value that was expected?

p0 = 428/(428 + 152) = 0.737931

If you round your answer of 0.737931 to three decimals you will

get 0.738.

Suppose that g(x)= f(x)+ 6. Which statement best compares the graph of g(x) with the graph of f(x)?

A. The graph of g(x) is the graph of f(x) shifted 6 units down.

B. The graph of g(x) is the graph of f(x) shifted to the right.

C. The graph of g(x) is the graph of f(x) shifted 6 units to the left.

D. The graph of g(x) is the graph of f(x) shifted 6 units up.

Answers

Answer:

D

Step-by-step explanation:

The + 6 moves it up 6 units.

The correct answer is (D) "The graph of g(x) is the graph of f(x) shifted 6 units up."

What is the function?

A relationship between a group of inputs and one output is referred to as a function. In plain English, a function is an association between inputs in which each input is connected to precisely one output. A domain, codomain, or range exists for every function. Typically, f(x), where x is the input, is used to represent a function.

When we add a constant to a function, such as in the case of g(x) = f(x) + 6, it will shift the graph of f(x) upward by 6 units.

This is because, for any value of x, the value of f(x) will be added to 6, resulting in a vertical shift of the entire graph.

Option (A) is incorrect because adding 6 to f(x) would shift the graph up, not down.

Option (B) is incorrect because adding a constant to a function does not cause it to shift horizontally.

Option (C) is incorrect because adding 6 to f(x) would shift the graph right, not left.

D. The graph of g(x) is the graph of f(x) shifted 6 units up. Adding a constant term to a function will shift the graph of the function vertically. In this case, adding 6 to f(x) will shift the graph of f(x) upward by 6 units, resulting in the graph of g(x).

Learn more about function here:

https://brainly.com/question/29633660

#SPJ7

Joe drives for 3 hours and covers 201 miles. In miles per hour, how fast was he driving?​

Answers

Answer:

67 mph

Step-by-step explanation:

201/3 = 67

international system of 89643092 in words​

Answers

Answer:

Eighty nine million six hundred forty three thousand ninety two

Step-by-step explanation:

89,643,092

=> Eighty nine million six hundred forty three thousand ninety two

What is the area of this figure?

Answers

Answer:

90km² only if it is parallelogram

Step-by-step explanation:

base = 9km

height=10km

area of parallelogram = b x h

=9km x 10km

=90km²

Answer:

A = 90km2

Step-by-step explanation:

Area of a rhombus is:

1. A = s x h (if given side and height)

2. A = 1/2 a x b (if given lengths of diagonals)

3. A = s^2 sin A (if given side and length)

Therefore from your problem, height and side is given thus, you'll use number 1

A = s x h

A = 9km x 10km = 90km2

Simplify:......................................................

Answers

Answer:

...

Step-by-step explanation:...

The cut off number is 2x-1

Determine whether the following event is mutually exclusive or not mutually exclusive.

Choosing a student who is a mathematics major or a business major from a nearby university to participate in a research study. (Assume that each student only has one major.)

Answers

Answer:

The event is mutually exclusive.

Step-by-step explanation:

Mutually exclusive events are events that cannot exist simultaneously.

Thus, events that are not mutually exclusive can exist simultaneously.

Since each student only has one major, a single student cannot be both a mathematics major and a business major.

So, the event is mutually exclusive.

Given points (-3;-6), G(3; -2) and H(6; 1); determine:
(a) The equation of line FG

Answers

Answer:

The equation of line FG is [tex]y = \frac{2}{3}x - 4[/tex]

Step-by-step explanation:

Equation of a line:

The equation of a line has the following format:

[tex]y = mx + b[/tex]

In which m is the slope and b is the y-intercept.

F(-3;-6), G(3; -2)

When we have two points, the slope is given by the change in y divided by the change in x. So

Change in y : -2 - (-6) = -2 + 6 = 4

Change in x: 3 - (-3) = 3 + 3 = 6

Slope: [tex]m = \frac{4}{6} = \frac{2}{3}[/tex]

So

[tex]y = \frac{2}{3}x + b[/tex]

Finding b:

(3; -2) means that when [tex]x = 3, y = -2[/tex]. We use this to find b.

[tex]y = \frac{2}{3}x + b[/tex]

[tex]-2 = \frac{2}{3}(3) + b[/tex]

[tex]2 + b = -2[/tex]

[tex]b = -4[/tex]

The equation of line FG is [tex]y = \frac{2}{3}x - 4[/tex]


State the x- and y-intercepts.
1. y-intercept?
2. x-intercepts?
3. Is this parabola positive or negative?

Answers

1. Y-Intercepts are where the graph crosses the y-axis. For a parabola, this occurs once, at (0, -3) in the graph shown.

2. X-Intercepts are where the graph crosses the x-axis. For a parabola, this occurs twice, at (-1, 0) and (3, 0) in the graph shown.

3. A parabola that is positive opens upwards, while a parabola that is negative opens downwards. Therefore, this parabola is positive.

Hope this helps!! :)

Heyy!! Can someone help me please!!

3 (5x + 2) - 2 (4x -4)

I don’t know what to doooo!!

Answers

Answer:

7x + 14

Step-by-step explanation:

the first thing to do is expand the parentheses/brackets.

3(5x + 2) -2(4x - 4) will be

3(5x) + 3(2) -2(4x) -2(-4)

= 15x + 6 - 8x + 8

collect like terms

15x - 8x + 6 + 8 = 7x + 14

the answer is 7x + 14

Answer:

3(5x+2)-2(4x-4)

15x+6-8x+8

15x-8x+6+8

7x+14

Which number's estimate written as a single digit times a power of 10 will have a negative exponent?

Answers

105 i hope this helps if not then i’m sorry

Please please help me please I really need help please just tap on picture and you will see the question

Answers

Answer:

No , it is not a right angle triangle

Step-by-step explanation:

according to the pythagoras theorem in right angled triangle sum of square of two sides is equal to the square of it's hypotenuse.

using pythagoras theorem

a^2 + b^2 = c^2

9^2 + 16^2 = 25^2

81 + 256 = 625

337 = 625

since sum of square of two smallest sides of a triangle is not equal to the square of it's hypotenuse it can be concluded that the given figure does not form right angle triangle.

your teacher for the discussion-based assessment.
In circle D, LEDH LEDG.
1. Determine the length of JG using
circle D.
Show your work and write out your
justification.
E
57"
9
Be prepared to answer questions
about additional angles, arcs and
segments from circle D.
F
D
J
H
669
G
2.
In OK, mZHKG=x+ 10 and
MZIKI = 3x - 22. Find m F).
74K
F
F
3. Find MLADB
OG with FA and FE tangent at A and E.
4. Find m2ABD
А
F
5. Find mzAFE
82
B
G
6. Find mLACE
E
1489
H Н
VOD

Answers

Answer:

3

Step-by-step explanation:

There are 17 books on a shelf. 8 of these books are new. the rest of them are used. (GIVING BRAINLEST TO BEST ANSWER) what is the ratio?​

Answers

A. 9 to 8
B. 17 to 9?
Other Questions
Someone started fishing at 15.30 am after fishing for 2hours 15 min he rested for half an hour he then continued until 11am how long did he fish? please help asap ill give brainliest What are the two types of addition compounds Write the equation of the line that passes through the points (6,-6) and (7,-4) Put your answer in fully reduced point-slope form, unless it is a vertical or horizontal line. Where can Alveoli be found? A. In the cell membrane. B. In the hypothalamus. C. In the liver. D. In the lungs. the meal costs $52.95 for 5 people. Jenna ordered a dessert for $2.50. if Jenna deducts this amount from the total how much will each of the other people pay for their meal Mrs. Brown, Mrs. White, and Mrs. Gray are a teacher, a doctor, and a lawyer, not necessarily in that order. Each has a horse. One horse is white, one is brown, and one is gray. From the following clues, determine the occupation of each woman and the color of her horse. No ones name is the same as the color of her horse. The teacher owns a brown horse. Mrs. Gray is a doctor. Palm Corp. uses the periodic inventory system. During its first year of operations, Palm made the following purchases (list in chronological order of acquisition): 20 units at $50 35 units at $40 85 units at $30 Sales for the year totaled 135 units, leaving 5 units on hand at the end of the year. Ending inventory using the average cost method is A. $ 150 C. $ 250 B. $ 177 D. $1,540 The Napoleonic Code reflected what ideas of the French Revolution? en que favorece la adaptacin a la biodiversidad? d) The Princess was allowed to climb trees.e)Hector lived a lonely life in the King's castle.Answer these questions in one or two words only.a) Who first discovered that the Princess had climbed up a tree? How would you describe the author's use of foreshadowing?Authors use foreshadowing to camouflage something that may occur later in the story.Authors use foreshadowing to hint about something that may occur earlier in the story.Authors use foreshadowing to hint about something that may occur later in the story.Authors use foreshadowing to hide something that may occur later in the story. we had a pot of tea. i drank 3/8 of the tea. after my father drank 2/3 of the remainder, 100 ml of tea is left inside the pot. what is the proportion of the total amount of tea? write your answer as a fraction. If using the method of completing the square to solve the quadratic equation22 + 2x + 16 = 0, which number would have to be added to "complete thesquare"? A cyclist travels 7 km from home to the park.She leaves home at 17:40 and arrives at the park at 17:55.What is her average speed in km/h? help,any question answer Which of the following employs the correct punctuation?I went to camp in the fourth grade; that was a long time ago.I went to camp in the fourth grade that was a long time ago.I went to camp in the fourth grade however, that was a long time ago.I went to camp in the fourth grade. A long time ago. Which of the pairs of functions and their inverses will always have a domain and range of all real numbers? Jason combines the two intermediate steps shown. 2 equations. First: upper N subscript 2 (g) plus upper O subscript 2 (g) right arrow 2 upper N upper O (g). Second: 2 upper N upper O (g) plus upper O subscript 2 (g) right arrow 2 upper N upper O subscript 2 (g). Which best describes what Jason should do with the oxygen molecules? 2. Round off the following numbers to two decimal places:b) 4,5873c) 32,095a) 56,3456d) 13,997